Question

Please make sure to explain, make sure answer is correct and neat. Thank you!

3. Show that the time taken by a particle moving along a curve y = y(x) with velocity, ds dt center on the x-axis. Hint: t = Jo x x from the point (0,0) to the point (1,1) is minimum if the curve is a circle having its 1 ds

0 0
Add a comment Improve this question Transcribed image text
Answer #1

ах y-y(x) dS 0 (0,0)Let the particle moves from 'O' to 'P' along the path y=y(x)

We consider a infinitesimal segment ds on the path y(x).

Futher , we assume the particle's velocity is constant while travelling the path ds.

From geometry we find ds= \sqrt{dx^2+dy^2}= [ \sqrt{1+(\frac{dy}{dx})^2}]dx=dx \sqrt{1+y'^2}

Where y'= \frac{dy}{dx}

Given the particle's velocity v=\frac{ds}{dt}=x

So the time taken by the particle to travel the distance ds is given by dt=\frac{ds}{v}= \frac{\sqrt{[1+y'^2}]dx}{x}

So the total time taken by the particle to travle from 'O' to 'P' along the path y=y(x) is given by

t= \int_{O}^{P}dt=\int_{O}^{P} \frac{ \sqrt{[1+y'^2}]dx}{x}

Or

t= \int_{O}^{P}fdx [ where f=\frac{ \sqrt{1+y'^2}}{x} ]

Now from Least Action principle, we know that 't' will be minimum if it satisfies  Euler-Lagrange differential equation i.e

\frac{\partial f}{\partial y}-\frac{d}{dx}(\frac{\partial f}{\partial y'})=0...........................(i0

Now we find

\frac{\partial f}{\partial y}=0

So equation (i) reduces to

\frac{d}{dx}(\frac{\partial f}{\partial y'})=0

Therefore we can write

\frac{\partial f}{\partial y'}= K, [ Where K is a constant]

Differentiating f with respect to y' , we get

\frac{\partial f}{\partial y'}=\frac{y'}{x \sqrt{1+y'^2}}

Hence we can write

\frac{y'}{x \sqrt{1+y'^2}}=K

Or

\frac{y'^2}{x^2 (1+y'^2)}=K^2

Or

y'^2=K^2x^2 (1+y'^2)

Or

y'^2(1-K^2x^2) =K^2x^2

Or

y'^2 =\frac{K^2x^2}{(1-K^2x^2)}

Or

y' =\frac{Kx}{\sqrt{(1-K^2x^2)}}

Or

\frac{dy}{dx} =\frac{Kx}{\sqrt{(1-K^2x^2)}}

or

dy =\frac{Kxdx}{\sqrt{(1-K^2x^2)}}

Integrating both sides we get

\int dy =\int \frac{Kxdx}{\sqrt{(1-K^2x^2)}}

Let 1-K^2x^2=v^2\Rightarrow -K^2xdx=vdv

So we get

y=-\frac{1}{K}\int \frac{vdv}{v}= -\frac{v}{K}+L [ 'L' is a constant ]

Or

y=-\frac{\sqrt{1-K^2x^2}}{K}+L

Squaring both sides we get

K^2(y-L)^2=1-K^2x^2

Or

K^2[(y-L)^2+x^2]=1.................................

Or

[(y-L)^2+x^2]=P.................[ where P=1/K^2

Now From the given points we find when x=0 , y=0

L=P

And when x=1, y=1

L=2, So K=2

So the equation of the trajectory is

x^2+(y-2)^2=2

So the time taken by the particle is minimum if the curve is a circle and the center lies on y axis

Add a comment
Know the answer?
Add Answer to:
Please make sure to explain, make sure answer is correct and neat. Thank you! 3. Show...
Your Answer:

Post as a guest

Your Name:

What's your source?

Earn Coins

Coins can be redeemed for fabulous gifts.

Not the answer you're looking for? Ask your own homework help question. Our experts will answer your question WITHIN MINUTES for Free.
Similar Homework Help Questions
  • Please explain all the steps needed to complete this problem. Thank you for your time. A...

    Please explain all the steps needed to complete this problem. Thank you for your time. A position-time graph for a particle moving along the x axis is shown in the figure. The divisions along the horizontal axis represent 1.75 s and the divisions along the vertical axis represent 3.0 m (a) Find the average velocity in the time interval t = 5.25 s to t = 14.00 s m/s (b) Determine the instantaneous velocity at t = 7.00 s (where...

  • 3. 8p] Show that the force field F(x,y, z) sin y, x cos y + cos z, -y sin z) is conservative and use this fact to evalu...

    3. 8p] Show that the force field F(x,y, z) sin y, x cos y + cos z, -y sin z) is conservative and use this fact to evaluate the work done by F in moving a particle with unit mass along the curve C with parametrization r(t (sin t, t, 2t), 0 <t<T/2. 4. 8p] A thin wire has the shape of a helix x = sin t, 0 < t < 27r. If the t, y = cos t,...

  • Please make sure answer is 100% clear and correct, as a lot of experts are getting...

    Please make sure answer is 100% clear and correct, as a lot of experts are getting this question wrong. I need a specific angle for the direction part of the question. Also make sure to use sig figs. Thank you. Question 1 4.30 nC of charge is uniformly distributed along a thin rod of length L 9.20 cm as shown in the figure. What is the magnitude and direction of the electric field at point P, a distancer 16.1 cm...

  • I lost in this I need help please thank you 10) [12;8] Let F =(x² -...

    I lost in this I need help please thank you 10) [12;8] Let F =(x² - y, x) and C is the boundary of the closed region that is the bounded by the y-axis and the left half of the circle x² + y2 = 4. Assume counterclockwise orientation. (a) Find the work done by this force field on a particle that moves along C, without using Green's Theorem (that is, do it as line integrals: be careful with how...

  • PLEASE MAKE SURE YOUR ANSWER IS CORRECT. THANK YOU. A ring of radius a has a...

    PLEASE MAKE SURE YOUR ANSWER IS CORRECT. THANK YOU. A ring of radius a has a charge distribution on it that varies as lambda(theta) = lambda_0 sin theta, as shown in the figure below. (a) What is the direction of the electric field at the center of the ring? upward radially outward radially inward downward (b) What is the magnitude of the field at the center of the ring? (Use the following as necessary: a, k, and lambda_0.)

  • Please answer all 3 question in neat handwriting. Thank you Find the particular solution for the...

    Please answer all 3 question in neat handwriting. Thank you Find the particular solution for the equation. dy = 8x3-9x? +7x: y=0 when x = 1 y = Determine the location of each local extremum of the function. f(x)=x? + 6x2 - 2 What is/are the local maximum/maxima? Select the correct choice below and, if necessary, fill in the answer box to complete your choice. O A. The local maximum/maxima is/are at x = (Use a comma to separate answers...

  • please write clearly on paper and show all steps and circle the correct answer. Thank you!...

    please write clearly on paper and show all steps and circle the correct answer. Thank you! Use implicit differentiation to the curve ex+2y = x + xy +5. Then find the slope of the curve at the point (-2,1).

  • please help me answer all 4, I need help. thank you Athin wire represented by the...

    please help me answer all 4, I need help. thank you Athin wire represented by the smooth curve C with a density (units of mass per longth) has a mass M ds. Find the mass of the wire c {xyxy=4x?, Os 55) with density px.y)-6+3xy. The mass of the wro is about (Round to one decimal place as needed.) Given the force field F, find the work required to move an object on the given oriented curve. F = (y....

  • PLEASE MAKE SURE ANSWER IS CORRECT PLEASE MAKE SURE ANSWER IS CORRECT Solve the system of...

    PLEASE MAKE SURE ANSWER IS CORRECT PLEASE MAKE SURE ANSWER IS CORRECT Solve the system of linear equations, using the Gauss-Jordan elimination method. (If there is no solution, enter NO SOLUTION. If there are infinitely many solutions, express your answer in terms of the parameters t and/or s.) x - 2y = 3 7x - 14y = 21 4x - 8y = 12 (x, y) =

  • I AM STRUGGLING HEAVILY ON THIS PROBLEM PLEASE HELP ME WITH THE CORRECT ANSWER AND NEAT...

    I AM STRUGGLING HEAVILY ON THIS PROBLEM PLEASE HELP ME WITH THE CORRECT ANSWER AND NEAT WORK PLEASE PLEASE AND THANK YOU Problem 1. (1 point) A steel ball weighing 128 pounds is suspended from a spring. This stretches the spring 178 feet. The ball is started in motion from the equilibrium position with a downward velocity of 6 feet per second. The air resistance (in pounds) of the moving ball numerically equals times its velocity (in feet per second)...

ADVERTISEMENT
Free Homework Help App
Download From Google Play
Scan Your Homework
to Get Instant Free Answers
Need Online Homework Help?
Ask a Question
Get Answers For Free
Most questions answered within 3 hours.
ADVERTISEMENT
ADVERTISEMENT
ADVERTISEMENT